LSAT and Law School Admissions Forum

Get expert LSAT preparation and law school admissions advice from PowerScore Test Preparation.

 silent7706
  • Posts: 42
  • Joined: Mar 26, 2019
|
#65032
P: "Since removing asbestos from buildings disturbs it, thereby releasing asbestos fibers, [posing health risk]"
C: "the gov't should not require removal of all asbestos insulation"

Hello, I was quick to narrow down between (B) and (E), and I ultimately picked (B) instead.

Can you please elaborate (B) vs (E)? Some people online suggest that part of fault with (B) is that it repeats the claim from stimuli that removing asbestos can pose healthy risk, because (B) states "asbestos can pose a health threat to workers who remove it.." However I'm not very convinced, because in strengthening a causal relationship A :arrow: B, we often use an example A :arrow: B to strengthen such a relationship. Perhaps, strengthening a causal relationship and is different from strengthening what we have here, and hence it is not a relevant comparison.

Thanks in advance.
 Jay Donnell
PowerScore Staff
  • PowerScore Staff
  • Posts: 144
  • Joined: Jan 09, 2019
|
#65327
Hi silent7706!

I think the most clear and useful factor in why B falls short here is that it brings up an additional factor involved with the risk borne by removing asbestos in that we are narrowing down to workers who are not using protective gear. Knowing that, we are then left to wonder how often workers do or don't have their (required) protective gear, and if they really face any danger at all while taking the proper protective measures.

Also, we are (whenever possible) drawn to answers with stronger logical force as attractive options in Strengthen questions, and the can in (B) also helps to dull its usable potency.

Between those two issues, B doesn't offer much power in strengthening the argument, whereas E highlights the idea that forced removal of asbestos and subsequent placement in a landfill might not be its final resting place, which compounds the negative health risks associated each time the asbestos is disturbed.

Hope that helps to clear it up!
User avatar
 anureet
  • Posts: 22
  • Joined: Aug 06, 2021
|
#91481
Isn't the first sentence suggesting a conditional relationship?

Asbestos causes harm -> disturbed and released into the environment

Now the second line is saying asbestos is disturbed and release into the environment, but we don't have any reason to believe that will cause harm (don't have that as a sufficient condition).

So based on this structure I thought, We need to prove that these conditions are sufficient to cause harm . Hence, I chose C.

Where did I go wrong in my approach?
 Adam Tyson
PowerScore Staff
  • PowerScore Staff
  • Posts: 5153
  • Joined: Apr 14, 2011
|
#91544
It was a good approach, Anureet, and if there had been an answer that said "all asbestos causes harm when disturbed," I think that would have been a good strengthen answer. The problem with answer C is that it only says that SOME asbestos causes MORE harm than other asbestos. So maybe there is a lot of harmless asbestos? That wouldn't be much help to us! Also, "greater" harm need not be "great" harm, since it is just a relative claim. So maybe the harm of leaving it in place is greater than the harm of removing it?

Answer E is much better because it tells us that disturbing it once may not be the end of it. The removed asbestos could continue to be a problem again and again every time it is disturbed! We cannot prove that disturbed asbestos will cause harm, because that would be a Mistaken Reversal, but if it can only be harmful when disturbed and is not harmful when not disturbed, that would certainly suggest that the best course of action is to never disturb it to avoid that risk. And we for sure don't want to disturb it over and over again!
User avatar
 treyrg
  • Posts: 12
  • Joined: Jun 18, 2021
|
#91724
Hi! I'm a little confused about the correct AC (E). I originally chose (C), but I didn't really like it in the first place. I just liked it more than (E) because I felt it made a little more of a connection between the premises and the conclusion than (E) did. I see now that this is not really the case, which would help me rule out (C). Since I was down to (C) and (E), I feel confident I, then, would have chosen (E), but I'd really prefer to get my thought process to the point where (E) sticks out to me as correct rather than choosing (E) because (C) sticks out as incorrect. I know this may not always be possible, but while practicing, that's obviously what I'm striving for.

For Strengthen questions, here's my basic methodology: find the relevant premise(s) and the conclusion, then find an AC that, specifically, strengthens the *argument* - so, I need something that makes the premises more likely to lead to the conclusion than they currently seem to given how the argument is written.

To do the above accurately, I need a good idea of how the argument is structured. Here's what I have:

P1: Health risk :arrow: Asbestos was disturbed and fibers were released
P2: Removal of asbestos :arrow: asbestos was disturbed
C: The government should not require the removal of all asbestos insulation.

This argument is obviously not great, but it's a little difficult for me to see how (E) strengthens it. I would probably want to say something more about how disturbing the asbestos makes it very likely for there to be a health risk because, as of now, we just know that if there is a health risk, then it's because the asbestos was disturbed. But the frequency for that happening is left open entirely, so it may happen very rarely or quite often. I thought this was the gap in reasoning.

I didn't find an AC that matched this very well, so I looked through them again with an open mind to see if there was an AC that at least did what I wanted (strengthened the argument) even though it didn't do it in the way I would have expected. While (C) doesn't feel particularly helpful (the donut analogy given above was very helpful in sorting out my thoughts there - thank you!), (E) doesn't either. Put more simply, I see how (E) might help to support the conclusion of the argument, but I'm having a difficult time seeing how it supports the *argument*.

I imagine I'm missing something here? Or, maybe I just have too high of expectations for a correct AC for Strengthen questions, and (E) being a relatively weaker AC makes it hard for me to see it as correct? I'm not sure, but any help would be appreciated! Sorry this is so long, but I thought that the more of my thought process you had detailed, the better you could help me understand what mistake I am making.

Thank you so much!
 Adam Tyson
PowerScore Staff
  • PowerScore Staff
  • Posts: 5153
  • Joined: Apr 14, 2011
|
#91748
I do think you are missing the manner in which answer E connects the premises more securely to the conclusion, treyrg. The evidence is that disturbing asbestos is the only cause of health risks related to it. The author concludes that the government should not require its removal, because that would disturb it and actually increase the risk. Basically, it's "leave it alone, it isn't harming anyone where it is, and you could make matters worse."

Answer E adds to that by indicating that removing it could make matters worse multiple times over. Not only would it be disturbed when it is removed, but it might be disturbed again, and again, and again if we remove it and bury it in a landfill. That's further connecting the evidence (disturbing it is the only source of risk) to the conclusion (don't require its removal). Imagine the original argument this way:

P: Disturbing it could be bad
P: Removing it disturbs it
C: Don't remove it

Now, here it is with answer E added:

P: Disturbing it could be bad
P: Removing it disturbs it
P: After you remove it, it could then be disturbed again multiple times
C: Don't remove it

Isn't the second argument stronger than the first, thanks to that extra premise?

Get the most out of your LSAT Prep Plus subscription.

Analyze and track your performance with our Testing and Analytics Package.